Xem bài viết đơn
Old 11-01-2018, 01:23 PM   #1
MATHSCOPE
Administrator

 
Tham gia ngày: Nov 2007
Bài gởi: 30
Thanks: 110
Thanked 183 Times in 68 Posts
VMO 2018, lời giải và bình luận

Bài 1. Cho dãy số $\left\{x_n\right\}_{n\in\mathbb Z^+}$ xác định bởi công thức truy hồi $x_1=2$ và
\[{x_{n + 1}} = \sqrt {{x_n} + 8} - \sqrt {{x_n} + 3}\quad\forall\,n\in\mathbb Z^+ .\]
  1. Chứng minh rằng dãy đã cho hội tụ và tính giới hạn.
  2. Chứng minh rằng
    \[n \le {x_1} + {x_2} + \ldots + {x_n} \le n + 1\quad\forall\,n\in\mathbb Z^+ .\]
link: http://mathscope.org/showthread.php?t=51561

Bài 2. Cho tam giác nhọn không cân $ABC$ với $D$ là một điểm trên cạnh $BC$ . Lấy điểm $E$ trên cạnh $AB$ và điểm $F$ trên cạnh $AC$ sao cho $\widehat{DEB}=\widehat{DFC}$. Các đường thẳng DF,DE lần lượt cắt $AB,AC$ tại $M,N$. Gọi $(I_1),(I_2)$ tương ứng là các đường tròn ngoại tiếp tam giác $DEM,DFN$. Kí hiệu $(J_1)$ là đường tiếp xúc trong với $(I_1)$ tại $D$ và tiếp xúc với $AB$ tại $K$, $(J_2)$ là đường tròn tiếp xúc trong với $(I_2)$ tại $D$ và tiếp xúc với $AC$ tại $H$, $P$ là giao điểm của $(I_1)$ và $(I_2)$, $Q$ là giao điểm của $(J_1)$ và $(J_2)$ ($P,Q$ khác $D$)
  1. Chứng minh $D,P,Q$ thẳng hàng.
  2. Đường tròn ngoại tiếp tam giác $AEF$ cắt đường tròn ngoại tiếp tam giác $AHK$ và đường thẳng $AQ$ lần lượt tại $G$ và $L$ ($G,L$ khác $A$).Chứng minh rằng tiếp tuyến tại $D$ của đường tròn ngoại tiếp tam giác $DQG$ cắt đường thẳng $EF$ tại một điểm nằm trên đường tròn ngoại tiếp tam giác $DLG$.
link: http://mathscope.org/showthread.php?t=51563


Bài 3. Mội nhà đầu tư có hai mảnh đất hình chữ nhật cùng kích thước $120m \times 100m$.
  1. Trên mảnh đất thứ nhất, nhà đầu tư muốn xây một ngôi nhà có nền hình chữ nhật có kích thước $25m \times 35m$ và xây bên ngoài $9$ bồn hoa hình tròn đường kính $5m$. Chứng minh rằng dù xây trước $9$ bồn hoa ở đâu thì trên phần đất còn lại vẫn đủ xây ngôi nhà đó.
  2. Trên mảnh đất thứ hai, nhà đầu tư muốn xây một hồ cá hình đa giác lồi sao cho từ một điểm bất kì trên phần đất còn lại có thể đi không quá $5m$ thì đến bờ hồ. Chứng minh rằng chu vi hồ không nhỏ hơn $(440-20\sqrt{2})m.$

link: http://mathscope.org/showthread.php?t=51564


Bài 4. Gọi $(C)$ là đồ thị hàm số $y=\sqrt[3]{x^2}$ trong mặt phẳng toạ độ $(Oxy)$. Một đường thẳng $(d)$ thay đổi cắt $(C)$ tại ba điểm phân biệt có hoành độ lần lượt là $x_1;\,x_2;\,x_3$.
  1. Chứng minh rằng đại lượng $\sqrt[3]{{\dfrac{{{x_1}{x_2}}}{{x_3^2}}}} + \sqrt[3]{{\dfrac{{{x_2}{x_3}}}{{x_1^2}}}} + \sqrt[3]{{\dfrac{{{x_3}{x_1}}}{{x_1^2}}}}$ là một hằng số.
  2. Chứng minh rằng
    \[\sqrt[3]{{\frac{{x_1^2}}{{{x_2}{x_3}}}}} + \sqrt[3]{{\frac{{x_2^2}}{{{x_3}{x_1}}}}} + \sqrt[3]{{\frac{{x_3^2}}{{{x_1}{x_2}}}}} < - \frac{{15}}{4}.\]

link: http://mathscope.org/showthread.php?t=51562

Bài 5. Cho các số nguyên dương $n$ và $d$. Xét tập hợp $S_{n}(d)$ gồm tất cả các bộ số có thứ tự $(x_1;...;x_d)$ thỏa mãn điều kiện sau:
  1. $x_{i}\in{1;2;...;n}$ với mọi chỉ số $1\leq i\leq d$.
  2. $x_{i}\neq x_{i+1}$ với mọi chỉ số $1\leq i\leq d-1$.
  3. Không tồn tại các chỉ số $1\leq i< j< k< l\leq d$ sao cho $x_i=x_k$ và $x_j=x_l$.

a)Tính số phần tử của tập hợp $S_{3}(5)$.

b)Chứng minh rằng tập hợp $S_{n}(d)$ khác rỗng khi và chỉ khi $d\leq 2n-1$.

link: http://forum.mathscope.org/showthread.php?t=51572

Bài 6. Cho dãy số $(x_n)$ xác định bởi $x_0=2,x_1=1$ và $x_{n+2}=x_{n+1}+x_{n}\left ( n\geq 0 \right )$

a)Với $n\geq 1$, chứng minh rằng nếu $x_n$ là số nguyên tố thì $n$ là số nguyên tố hoặc $n$ không có ước nguyên tố lẻ.

b)Tìm các cặp số nguyên không âm $(m,n)$ sao cho $x_n$ chia hết cho $x_m$.
link: http://forum.mathscope.org/showthread.php?t=51571

Bài 7. Cho tam giác nhọn không cân $ABC$ có trọng tâm $G$ nội tiếp đường tròn $(O)$. Gọi $H_a,H_b,H_c$ lần lượt là chân đường cao hạ từ các đỉnh $A,B,C$ của tam giác $ABC$ và $D,E,F$ lần lượt là trung điểm các cạnh $BC,CA,AB$. Các tia $GH_a,GH_b,GH_c$ lần lượt cắt $(O)$ tại các điểm $X,Y,Z$

a)Chứng minh rằng đường tròn ngoại tiếp tam giác $XCE$ đi qua trung điểm của đoạn thẳng $BH$

b)Gọi $M,N,P$ tương ứng là trung điểm các đoạn thẳng $AX,BY,CZ$. Chứng minh rằng các đường thẳng $DM,EN,FP$ đồng quy.

link: http://forum.mathscope.org/showthrea...924#post212924

PS. Có thể thảo luận ngay tại đây, hoặc theo link từng bài.
[RIGHT][I][B]Nguồn: MathScope.ORG[/B][/I][/RIGHT]
 
MATHSCOPE is offline   Trả Lời Với Trích Dẫn
The Following 2 Users Say Thank You to MATHSCOPE For This Useful Post:
buratinogigle (13-01-2018), vnt.hnue (12-01-2018)
 
[page compression: 13.25 k/14.34 k (7.59%)]